User avatar
 
LSAT-Chang
Thanks Received: 38
Atticus Finch
Atticus Finch
 
Posts: 479
Joined: June 03rd, 2011
 
 
trophy
Most Thankful
trophy
First Responder
 

Q1 - The development of new inventions

by LSAT-Chang Mon Jul 11, 2011 10:59 am

Is (C) wrong because we are not concerned with the "costs incurred in applying for patent rights"? I was debating between (A) and (C) and I did pick (A) but it was out of guess between the two. I thought we might have to assume that the costs to apply for patent rights are insignificant compared to the financial benefit of holding the patent rights because if we negate it, then we get that those costs are "significant" so it would destroy the conclusion since then, it won't be so important to continue to grant patent rights if inventors are paying a lot for applying for these patent rights and don't have much financial benefit. Does this make sense?

Or maybe it's just out of scope. But I feel like with these necessary assumption questions, some of the answers are something that seems really bizarre and out of scope at first, but turns out that we actually need to assume them. So I started "looking out" for some answer choices that seemed way out of scope at first.. is this still a good strategy?
User avatar
 
demetri.blaisdell
Thanks Received: 161
LSAT Geek
 
Posts: 198
Joined: January 26th, 2011
 
This post thanked 2 times.
 
 

Re: Q1 - The development of new inventions

by demetri.blaisdell Tue Jul 12, 2011 12:11 pm

Generally speaking, correct answers to necessary assumption questions show a lot of variety. For that reason, we caution our students about dismissing answer choices as out of scope without really understanding how they might affect the argument. I don’t tell my students to focus on seemingly out of scope answer choices, however.

The core of the argument in this question is:

Without patents, inventors won’t make money → Continue granting patent rights or else no new inventions

So what’s the gap? One competing possibility here is that inventors could be eccentric enough to keep on inventing things even if they knew they weren’t going to make money. Or maybe most inventions are accidental. These possibilities might seem unlikely, but we don’t have any information to connect the premise that inventors won’t make money without patents with the conclusion that they will stop inventing. (A) gives us exactly what we need. Money is the only reason people invent things. Put it in to the argument above and see how it closes the gap we identified.

(B) is a premise booster. It tries to tell us more information about inventors making money"”specifically that they make more than manufacturers. We already know they won’t make money without patents; we’re looking for something that tells us whether or not money is why they invent.

(C) is another premise booster. We have to accept the premise as true when it says that inventors won’t make money without patents to protect their inventions. Telling us that their application costs are insignificant compared to their reward is unnecessary information. Apply the negation test: Application costs are significant compared to the benefit of the patent. Significant does not mean greater than. All this tells me is that inventors are making less money on each invention. This does not help me address the gap of why inventors invent.

(D) is out of scope. This answer choice cannot be related to either the premise or the conclusion.

(E) is also out of scope. The argument does not depend on this assumption.

Let me know if this clears up your (A) versus (C) problem.

Demetri
User avatar
 
LSAT-Chang
Thanks Received: 38
Atticus Finch
Atticus Finch
 
Posts: 479
Joined: June 03rd, 2011
 
 
trophy
Most Thankful
trophy
First Responder
 

Re: Q1 - Patent Rights

by LSAT-Chang Tue Jul 12, 2011 4:28 pm

demetri.blaisdell Wrote:Generally speaking, correct answers to necessary assumption questions show a lot of variety. For that reason, we caution our students about dismissing answer choices as out of scope without really understanding how they might affect the argument. I don’t tell my students to focus on seemingly out of scope answer choices, however.

The core of the argument in this question is:

Without patents, inventors won’t make money → Continue granting patent rights or else no new inventions

So what’s the gap? One competing possibility here is that inventors could be eccentric enough to keep on inventing things even if they knew they weren’t going to make money. Or maybe most inventions are accidental. These possibilities might seem unlikely, but we don’t have any information to connect the premise that inventors won’t make money without patents with the conclusion that they will stop inventing. (A) gives us exactly what we need. Money is the only reason people invent things. Put it in to the argument above and see how it closes the gap we identified.

(B) is a premise booster. It tries to tell us more information about inventors making money"”specifically that they make more than manufacturers. We already know they won’t make money without patents; we’re looking for something that tells us whether or not money is why they invent.

(C) is another premise booster. We have to accept the premise as true when it says that inventors won’t make money without patents to protect their inventions. Telling us that their application costs are insignificant compared to their reward is unnecessary information. Apply the negation test: Application costs are significant compared to the benefit of the patent. Significant does not mean greater than. All this tells me is that inventors are making less money on each invention. This does not help me address the gap of why inventors invent.

(D) is out of scope. This answer choice cannot be related to either the premise or the conclusion.

(E) is also out of scope. The argument does not depend on this assumption.

Let me know if this clears up your (A) versus (C) problem.

Demetri


Thanks so much! A major problem I have with assumption questions is that I always end up picking answer choices that are "premise boosters". I really can't seem to figure out why that is the case. Do you have any advice as to how to possibly not fall into this trap?
User avatar
 
demetri.blaisdell
Thanks Received: 161
LSAT Geek
 
Posts: 198
Joined: January 26th, 2011
 
 
 

Re: Q1 - Patent Rights

by demetri.blaisdell Thu Jul 14, 2011 10:33 am

If you are often choosing premise boosters on assumption questions, it could be because you aren't fully understanding the gap in the argument before you look at the answers. While you may not be able to predict necessary assumptions (which can often seem out of scope at first), finding the gap in the argument will help you understand the premise(s) and conclusion.

Take an assumption practice set out of the book and do it untimed. Before you even look at the answer choices (cover them if you have to), write out the argument core and identify the gap. You don't have to guess the assumption perfectly, but focusing on the gap between the premise and the conclusion will prepare you for an answer choice that addresses that gap.

I hope that helps you improve. Let me know if you are still having trouble and I will give you some other ways to practice these skills.
 
weiwu0221
Thanks Received: 0
Vinny Gambini
Vinny Gambini
 
Posts: 4
Joined: October 05th, 2011
 
 
 

Re: Q1 - Patent Rights

by weiwu0221 Sat Oct 22, 2011 3:40 am

demetri.blaisdell Wrote:Generally speaking, correct answers to necessary assumption questions show a lot of variety. For that reason, we caution our students about dismissing answer choices as out of scope without really understanding how they might affect the argument. I don’t tell my students to focus on seemingly out of scope answer choices, however.

The core of the argument in this question is:

Without patents, inventors won’t make money → Continue granting patent rights or else no new inventions

So what’s the gap? One competing possibility here is that inventors could be eccentric enough to keep on inventing things even if they knew they weren’t going to make money. Or maybe most inventions are accidental. These possibilities might seem unlikely, but we don’t have any information to connect the premise that inventors won’t make money without patents with the conclusion that they will stop inventing. (A) gives us exactly what we need. Money is the only reason people invent things. Put it in to the argument above and see how it closes the gap we identified.

(B) is a premise booster. It tries to tell us more information about inventors making money"”specifically that they make more than manufacturers. We already know they won’t make money without patents; we’re looking for something that tells us whether or not money is why they invent.

(C) is another premise booster. We have to accept the premise as true when it says that inventors won’t make money without patents to protect their inventions. Telling us that their application costs are insignificant compared to their reward is unnecessary information. Apply the negation test: Application costs are significant compared to the benefit of the patent. Significant does not mean greater than. All this tells me is that inventors are making less money on each invention. This does not help me address the gap of why inventors invent.

(D) is out of scope. This answer choice cannot be related to either the premise or the conclusion.

(E) is also out of scope. The argument does not depend on this assumption.

Let me know if this clears up your (A) versus (C) problem.

Demetri


Thank you for your explanation.
(A) is the best answer available, but it looks like a sufficient condition rather than a neccessary one for the modifier "only". Because it preludes all other possibilities, it's more than neccessary. What do you think about that?
 
djy_neworiental
Thanks Received: 0
Vinny Gambini
Vinny Gambini
 
Posts: 3
Joined: October 25th, 2011
 
 
 

Re: Q1 - Patent Rights

by djy_neworiental Sat Nov 05, 2011 1:17 pm

I have the same question. I understand A can fill the gap but it seems more like a sufficent assumption because of the word "only". So if I negate A and financial reward is not the only incentive, the conc seems to be ok. Because, you know, what if financial reward is the primary incentive?
and I chose c because when I negate c, I ask myself what if the cost is more significant than the future profit? the conc seems to fall apart.
I find assumption question very difficult for I dont know when it is out-of-scope and when the answer eliminates a potential possibility/problem. Can anyone give me some advice? Thanks.
weiwu0221 Wrote:
demetri.blaisdell Wrote:Generally speaking, correct answers to necessary assumption questions show a lot of variety. For that reason, we caution our students about dismissing answer choices as out of scope without really understanding how they might affect the argument. I don’t tell my students to focus on seemingly out of scope answer choices, however.

The core of the argument in this question is:

Without patents, inventors won’t make money → Continue granting patent rights or else no new inventions

So what’s the gap? One competing possibility here is that inventors could be eccentric enough to keep on inventing things even if they knew they weren’t going to make money. Or maybe most inventions are accidental. These possibilities might seem unlikely, but we don’t have any information to connect the premise that inventors won’t make money without patents with the conclusion that they will stop inventing. (A) gives us exactly what we need. Money is the only reason people invent things. Put it in to the argument above and see how it closes the gap we identified.

(B) is a premise booster. It tries to tell us more information about inventors making money"”specifically that they make more than manufacturers. We already know they won’t make money without patents; we’re looking for something that tells us whether or not money is why they invent.

(C) is another premise booster. We have to accept the premise as true when it says that inventors won’t make money without patents to protect their inventions. Telling us that their application costs are insignificant compared to their reward is unnecessary information. Apply the negation test: Application costs are significant compared to the benefit of the patent. Significant does not mean greater than. All this tells me is that inventors are making less money on each invention. This does not help me address the gap of why inventors invent.

(D) is out of scope. This answer choice cannot be related to either the premise or the conclusion.

(E) is also out of scope. The argument does not depend on this assumption.

Let me know if this clears up your (A) versus (C) problem.

Demetri


Thank you for your explanation.
(A) is the best answer available, but it looks like a sufficient condition rather than a neccessary one for the modifier "only". Because it preludes all other possibilities, it's more than neccessary. What do you think about that?
 
csunnerberg13
Thanks Received: 24
Elle Woods
Elle Woods
 
Posts: 62
Joined: April 10th, 2013
 
 
 

Re: Q1 - The development of new inventions

by csunnerberg13 Tue Aug 27, 2013 5:23 pm

Why is A not incorrect as a sufficient assumption??
User avatar
 
WaltGrace1983
Thanks Received: 207
Atticus Finch
Atticus Finch
 
Posts: 837
Joined: March 30th, 2013
 
 
trophy
Most Thanked
trophy
Most Thankful
trophy
First Responder
 

Re: Q1 - Patent Rights

by WaltGrace1983 Tue Jul 01, 2014 2:11 pm

weiwu0221 Wrote:(A) is the best answer available, but it looks like a sufficient condition rather than a neccessary one for the modifier "only". Because it preludes all other possibilities, it's more than neccessary. What do you think about that?


~(patent rights) → ~(financial incentive)
+
~(parent rights)
→
~(engagement in original development / new inventions)

Just because something is necessary doesn't mean that it is NOT sufficient also. The question of "necessary or sufficient?," in true LSAT fashion, does not preclude the possibility of both. I would argue that (A) is a sufficient condition. If we say that ~(financial incentive) → ~(motivation to develop new inventions) then the argument seems to follow logically with no holes.

However, it is also necessary. If financial reward is NOT the ONLY incentive, then that means there are other incentives. Could the conclusion still be true? Oh yea! Does the conclusion follow from the premises? Absolutely not. Why? Because sticking in this negated answer choice (A) would make you think, "why did you even bring up the premises then, LSAT writer?!" This is a subtle indication that the conclusion simply doesn't follow logically from the premises.

djy_neworiental Wrote:I have the same question. I understand A can fill the gap but it seems more like a sufficent assumption because of the word "only". So if I negate A and financial reward is not the only incentive, the conc seems to be ok. Because, you know, what if financial reward is the primary incentive?

and I chose c because when I negate c, I ask myself what if the cost is more significant than the future profit? the conc seems to fall apart.


(1) see above. Necessary and sufficient are not exclusive terms.

(2) The conclusion may seem to be "okay," as in it could still be true, but the point is that the conclusion wouldn't FOLLOW from the PREMISES. That is what a necessary assumption is all about. If you negate something and the conclusion seems to not make all that much sense, you are doing it right!

(3) When you arrived at (C) you seemed to be committing the very same flaw this question is testing you on. You seemed to have assumed that "financial incentive" was critical to the development of inventions. It may not be.

csunnerberg13 Wrote:Why is A not incorrect as a sufficient assumption??


See above!

Hope that's helpful!